음함수 정리: 두 판 사이의 차이

내용 삭제됨 내용 추가됨
편집 요약 없음
70번째 줄:
* <math>\operatorname B(\mathbf x',\delta_4)\subseteq\operatorname B(\mathbf a,\delta_2)</math>
* 임의의 <math>\mathbf x\in\operatorname B(\mathbf x',\delta_4)</math>에 대하여, <math>h(\mathbf x)\in\operatorname B(b,\delta_1)</math>
따라서, 임의의 <math>\mathbf x\in\operatorname B(\mathbf x',\delta_4)</math>에 대하여, <math>h(\mathbf x)=g(\mathbf x)</math>이다. 즉, <math>A</math>는 <math>\operatorname B(\mathbf a,\delta_2)</math>의 열린집합이다. 또한 임의의 <math>\mathbf x'\in A'\cap\operatorname B(\mathbf a,\delta_2)</math>에 대하여, <math>g,h</math>의 연속성에 의하여 <math>\mathbf x'\in A</math>이다. 즉, <math>A</math>는 <math>\operatorname B(\mathbf a,\delta_2)</math>의 닫힌집합이다. <math>\operatorname B(\mathbf a,\delta_2)</math>는 [[연결 집합]]이며, 또한 <math>A\ne\varnothing</math>이므로, <math>A=\operatorname B(\mathbf a,\delta_2)</math>이다. 즉, 임의의 <math>\mathbf x\in\operatorname B(\mathbf a,\delta_2)</math>에 대하여, <math>g(\mathbf x)=h(\mathbf x)</math>이다. 이제 <math>g</math>의 연속 미분 가능성과 도함수 공식을 증명하자. 임의의 <math>j\in\{1,\dots,n\}</math> 및 <math>\mathbf x',\mathbf x'+\Delta x_j\mathbf e_j\in\operatorname B(\mathbf a,\delta_2)</math>에 대하여, [[평균값다음과 정리]]에 따라 다음을 만족시키는같이 표기하자.
:<math>0<\thetaDelta y(\mathbf x',\mathbf x'+\Delta x_j\mathbf e_j)<1=g(\mathbf x'+\Delta x_j\mathbf e_j)-g(\mathbf x')</math>가 존재한다.
그러면 [[평균값 정리]]에 따라 다음을 만족시키는 <math>0<\theta(\mathbf x',\mathbf x'+\Delta x_j\mathbf e_j)<1</math>가 존재한다.
:<math>\begin{align}0
&=f(\mathbf x'+\Delta x_j\mathbf e_j,g(\mathbf x')+\Delta x_j\mathbf e_j)y)-f(\mathbf x',g(\mathbf x'))\\
&=
&=\frac{\partial f}{\partial x_j}(\mathbf x'+\theta(\Delta x_j\mathbf x'e_j,\mathbf g(\mathbf x')+\theta\Delta x_j\mathbf e_jy)\Delta x_j\mathbf e_j)+
\Delta x_j+
\frac{\partial f}{\partial y}(\mathbf x'+\theta(\Delta x_j\mathbf x'e_j,\mathbf g(\mathbf x')+\theta\Delta x_j\mathbf e_jy)\Delta x_j\mathbf e_j)y
(g(\mathbf x'+\Delta x_j\mathbf e_j)-g(\mathbf x'))
\end{align}</math>
따라서, 다음이 성립한다.
:<math>\begin{align}\frac{\partial g}{\partial x_j}(\mathbf x')
&=\lim_{\Delta x_j\to\mathbf 0}\frac{g(\mathbf x'+\Delta x_j\mathbf e_j)-g(\mathbf x')y}{\Delta x_j}\\
&=\lim_{\Delta x_j\to\mathbf 0}\left(
-\frac{\partial f}{\partial x_j}(\mathbf x'+\theta(\Delta x_j\mathbf x'e_j,\mathbf g(\mathbf x')+\theta\Delta x_j\mathbf e_jy)\Delta x_j\mathbf e_j,bigg/
g(\mathbffrac{\partial x'+f}{\thetapartial y}(\mathbf x',+\mathbf x'+theta\Delta x_j\mathbf e_j),\Deltamathbf x_jg(\mathbf e_j)x')+\theta\bigg/Delta y)
\frac{\partial f}{\partial y}(\mathbf x'+\theta(\mathbf x',\mathbf x'+\Delta x_j\mathbf e_j)\Delta x_j\mathbf e_j,
g(\mathbf x'+\theta(\mathbf x',\mathbf x'+\Delta x_j\mathbf e_j)\Delta x_j\mathbf e_j))
\right)\\
&=-
&=-\frac{\partial f}{\partial x_j}(\mathbf x',g(\mathbf x'))\bigg/
(g(\mathbffrac{\partial x'+f}{\Deltapartial x_j}(\mathbf e_j)-x',g(\mathbf x'))\bigg/
\frac{\partial f}{\partial y}(\mathbf x',g(\mathbf x'))
\end{align}</math>